#33 Rosh Review

Pataasin ang iyong marka sa homework at exams ngayon gamit ang Quizwiz!

Question: Aside from BUN/Creatinine ratio, what other lab findings are consistent with prerenal acute renal failure?

Answer: A Urine Sodium <20 mEq/L and a Fraction of Excreted Na (FENa) <1% Rapid Review Acute Urinary Retention Obstruction (men: BPH, women: prolapse/tumor) Neurogenic (diabetes) Drugs (α-agonists) Rx: Foley

Question: What is the potential harmful outcome of untreated group A strep pharyngitis?

Answer: Acute rheumatic fever.

Question: At what age should discussion about prostate cancer screening occur in African-American men or those with a family history of prostate cancer?

Answer: 45 years.

A 42-year-old man presents to the Emergency Department with acute onset of severe shortness of breath. He was recently discharged from the hospital with endocarditis. Vital signs include temperature 37.6oC, HR 110, BP 110/50, RR 24, Pox 93%. On examination, he is ill appearing with a new harsh apical systolic murmur and bibasilar rales. Which of the following is the most likely diagnosis? Acute mitral regurgitation Aortic regurgitation Health care associated pneumonia Pulmonary embolism

Correct Answer ( A ) Explanation: Acute mitral regurgitation presents with acute onset of severe dyspnea and signs of pulmonary edema. Patients can rapidly decompensate into cardiogenic shock or cardiac arrest. A harsh systolic apical murmur and S4 gallop are present. It is most commonly due to rupture of the papillary muscle or chordae tendinae secondary to myocardial infarction. Other causes include rupture due to infective endocarditis, blunt chest trauma or spontaneous rupture. Chest x-ray will show signs of pulmonary edema with a normal heart size (unless the patient has chronic ischemic or valvular heart disease). Echocardiography will confirm the diagnosis and assess the degree of regurgitation. Emergency Department management includes oxygen and positive pressure ventilation as needed for respiratory failure. Nitrates and diuretics are indicated for treatment of pulmonary edema. An intra-aortic balloon pump may be needed for stabilization of the hypotensive patient until definitive treatment with surgery. Emergent cardiology and surgical consultation is imperative. Aortic regurgitation (B) can also present with severe dyspnea but is associated with a high pitched blowing diastolic murmur. Health care associated pneumonia (C) typically presents with fever, cough and findings of consolidation on lung exam. Although dyspnea may be severe and acute in onset in patients with pulmonary embolism (D), they do not typically present with a new heart murmur.

Question: True or false: fibromyalgia is more common in men?

Answer: False. Rapid Review Fibromyalgia Patient will be a woman Complaining of widespread musculoskeletal pain for > 3 months, non-restorative sleep and generalized fatigue PE will show tenderness at > 9 of 18 anatomic sites ("trigger points") Labs will be normal Diagnosis is made clinically Treatment is education, antidepressants, avoid opioids

Question: What syndrome can be seen when pelvic inflammatory disease spreads to the abdomen?

Answer: Fitz-Hugh-Curtis syndrome. Rapid Review Pelvic Inflammatory Disease (PID) Patient will be a Female With a history of multiple sexual partners or unprotected sex or both Complaining of lower abdominal pain, cervical motion tenderness, painful sexual intercourse PE will show mucopurulent cervical discharge, "Chandelier sign" Most commonly caused by Chlaymidia Treatment is ceftriaxone + doxycycline Comments: Fitz-Hugh-Curtis syndrome: perihepatitis + PID

Question: What is the value needed on culture for a clean catch specimen in order to diagnose a urinary tract infection?

Answer: Greater than 100,000 colony forming units (CFU).

Question: What constitutes a positive urine culture in a catheterized specimen?

Answer: Growth of >10,000 colony forming units (CFUs). Rapid Review Acute Pyelonephritis Patient will be complaining of fever, dysuria, and flank pain PE will show CVA tenderness Urinalysis will show leukocyte esterase, nitrites and microscopy will show WBCs. Most commonly caused by Escherichia coli Treatment is ciprofloxacin Inpatient/pregnant: ampicillin/gentamycin or 3rd gen cephalosporin

Question: What is the most common cause of adult epiglottitis?

Answer: H. influenzae type B is the most commonly identified bacterial pathogen and is associated with severe disease. However, very few patients actually have the etiologic organism identified. Rapid Review Epiglottitis Patient will be complaining of rapid onset of fever and dysphagia PE will show patient leaning forward, drooling, inspiratory stridor Imaging will show "thumbprint" sign Most commonly caused by H. influenzae, Streptococcus Treatment is IV antibiotics and airway management

Question: What medication is commonly used in combative patients with a diagnosis of delirium?

Answer: Haloperidol.

Question: What is the most common etiology of onychomycosis?

Answer: Dermatophyte infection. Rapid Review Onychomycosis Patient will be complaining of thickened and discolored toenails Diagnosis is made by KOH preparation of nail scraping Treatment is oral terbinafine Comments: Serum aminotransferases should be monitored before starting treatment with terbinafine and during the treatment due to hepatotoxicity

Question: What is the treatment for erysipelas?

Answer: A parenteral antistreptococcal antibiotic such as ceftriaxone or cefazolin. Rapid Review Erysipelas S. pyogenes Fever, chills → painful rash Face Bright red color, sharp margins Consider admission in all patients IV ABX

Question: When is emergent orthopedic consultation required for carpal tunnel syndrome?

Answer: Acute onset of symptoms over hours suggests the presence of acute median nerve compression from fracture or infection and requires emergent decompression. Rapid Review Carpal Tunnel Syndrome Patient with a history of extensive wrist usage such as typing Complaining of pain and numbness in the first, second, and third digits, especially at night PE will show Phalen's sign: reproduction of symptoms with wrist hyperflexion, Tinel's sign: reproduction of symptoms with percussion over carpal tunnel Most commonly caused by median nerve compression Treatment is NSAIDs, volar splint in neutral position

Question: What drug class is considered first-line for the treatment of BPH?

Answer: Alpha-adrenergic receptor blockers. Rapid Review Benign Prostatic Hyperplasia (BPH) Patient will be older Complaining of Hesitancy, Intermittence/Incontinence, Frequency/Fullness, Urgency, Nocturia (HI FUN) PE will show a soft, smooth and mobile prostate without any nodules or indurations Diagnosis is made by digital rectal exam Most commonly caused by stromal and epithelial cell growth in the transitional zone of the prostate Treatment is α-blockers, 5α-reductase inhibitors, surgery (TURP)

Question: What does hemoglobin A1c represent?

Answer: Although the A1C reflects mean blood glucose over the entire 120-day life span of the red blood cell, it correlates best with mean blood glucose over the previous 8 to 12 weeks. Rapid Review Diabetes Mellitus Type 1 Autoimmune destruction of β cells < 30 years old Polyuria, polydipsia, weight loss DKA Insulin always necessary

Question: What serum test should be obtained in all patients with an overdose regardless of agent?

Answer: An acetaminophen level. Acetaminophen overdose can present without symptoms early on, is a common drug in many combined medications and can have devastating complications if not caught and treated early. Rapid Review Suicide Protective factors: marriage, pregnancy RFs (SAD PERSONS): Sex (male), Age (teenager or elderly), Depression, Previous attempt, Ethanol/drug use, Rational thinking loss, Sickness (medical illness, 3 or more prescription medications), Organized plan, No Social support, Stated future attempt Most completed suicides involve firearms Most attempted suicides involve ingestions (MC: antidepressants) All overdose patients: obtain acetaminophen level

Question: Other than cardiac ischemia or infarct, what are some other cardiac causes of chest pain?

Answer: Aortic dissection, pericarditis and myopericarditis. Rapid Review Ischemic Heart Disease Leading cause of death in the USA in both men and women RFs: family hx, smoking, HTN, DM, cholesterol, male, age >55 Atypical presentations occur in up to one third of patients Chest pain, diaphoresis, nausea, hiccups, radiation to shoulder/jaw/back Elderly, diabetics, females, history of stroke or heart failure: increased risk for atypical presentation ECG: hyperacute T waves earliest sign of MI Up to 50% of ECGs are normal or nonspecific Troponin I most specific for cardiac injury Nitroglycerin contraindicated in right ventricular MI and recent sildenafil use Aspirin, nitrates, clopidogrel, heparin, and beta-blockers (within 24 hours) Preferred treatment for AMI: angioplasty or stenting

Question: What is the last resort option for recalcitrant paraphimosis?

Answer: Circumcision. Rapid Review Paraphimosis Inability to pull retracted foreksin back over glans Vascular emergency Initial rx: manual reduction with firm pressure for 5-10 minutes Dorsal slit if reduction unsuccessful

Question: What is the difference between condyloma acuminatum and condyloma lata?

Answer: Condyloma acuminatum are broad-based, pedunculated, cauliflower-like warts caused by human papillomavirus. Condyloma lata are broad-based, flat, moist papules due to Treponema pallidum.

Question: What toxin does hydroxocobalamin act as an antidote for?

Answer: Cyanide Rapid Review Vitamin Deficiencies A: vision loss, dry skin, growth retardation B1: alcoholism, malnutrition, Wernicke-Korsakoff syndrome B2: Cheliosis, Corneal vascularization (the 2 Cs of B2) B3: dermatitis, dementia, diarrhea, corn based diet B6: sideroblastic anemia, convulsions, peripheral neuropathy, INH use B12: megaloblastic anemia + neurologic symptoms, hypersegmented neutrophils C: scurvy (↑ bleeding, anemia, loose teeth) D: rickets (children), osteomalacia, tetany E: anemia, peripheral neuropathy, ataxia K: ↑ bleeding, ↑ PTT, normal BT

Question: What is the main benefit of steroid treatment in optic neuritis?

Answer: Delay of onset of multiple sclerosis. Rapid Review Optic Neuritis Idiopathic > MS Unilateral vision loss Pain with EOM Afferent pupillary defect Negative exam or optic disk swelling MRI to r/o MS Steroids

Question: True or false: women who have had a hysterectomy with removal of the cervix not related to cervical cancer should be screened for cervical cancer?

Answer: False.

Question: Is Homan's sign a reliable examination finding in the evaluation of deep venous thrombosis?

Answer: Homan's sign is pain in the calf with dorsiflexion of the ankle. It is an insensitive and nonspecific sign of a deep venous thrombosis. Rapid Review Pulmonary Embolism 95% arise from deep leg veins Sudden onset of symptoms in 50% SOB, CP, tachypnea ECG: sinus tachycardia, nonspecific ST-T changes, right heart strain, S1Q3T3 (classic finding) CXR: nonspecific abnormalities, Hampton's hump (pleural-based wedge infarct), Westermark's sign (vascular cut-off sign) V/Q scan: usually nondiagnostic Low clinical suspicion: negative D-dimer excludes PE Dx of choice: CTPA Rx: anticoagulation, thrombolytics (if HD unstable), embolectomy (last resort)

Question: Which underlying medical conditions are common in patients who develop necrotizing (malignant) otitis externa?

Answer: Immunocompromised patients such as diabetic patients, patients with acquired immunodeficiency syndrome (AIDS), and those receiving chemotherapy. Rapid Review Otitis Externa Patient with a history of swimming or moisture exposure Complaining of malodorous discharge and pruritus PE will show pain with palpation of tragus/pinna Most commonly caused by Pseudomonas aeruginosa Treatment is topical antimicrobials with or without steroids Comments: Necrotizing otitis externa - a complication seen in diabetics/immunocompromised

Question: What is the first-line treatment for an acute attack of gout?

Answer: Indomethicin.

Question: What anatomic structure delineates internal hemorrhoids?

Answer: Internal hemorrhoids originate above the dentate line. Rapid Review Hemorrhoids Internal Proximal to dentate line Painless bleeding External Below dentate line Visible Painful Initial rx: WASH (Water (sitz bath), Analgesics, Stool softeners, High fiber diet) External thrombosed hemorrhoid presenting within 48 - 72 hours rx: excision with elliptical incision

Question: What is Levine's sign?

Answer: It is when a patient describes their chest pain by holding their clenched fist over their chest. It is considered to be a sign of ischemic chest pain. Rapid Review Acute Bronchitis Infection of conducting airways, inflammatory/exudate, bronchospasms Most commonly caused by viruses Cough = hallmark of acute bronchitis, <1week Symptomatic treatment and bronchodilators Abx not needed

Question: What effect does finasteride have on the PSA level?

Answer: It lowers PSA levels by about 50% after 6 months of use. Rapid Review Benign Prostatic Hyperplasia (BPH) Patient will be older Complaining of Hesitancy, Intermittence/Incontinence, Frequency/Fullness, Urgency, Nocturia (HI FUN) PE will show a soft, smooth and mobile prostate without any nodules or indurations Diagnosis is made by digital rectal exam Most commonly caused by stromal and epithelial cell growth in the transitional zone of the prostate Treatment is α-blockers, 5α-reductase inhibitors, surgery (TURP)

Question: Traumatic femoral neuropathy may result in which motor symptoms?

Answer: Knee extension (quadriceps) weakness. Rapid Review Saphenous Nerve Neuropathy Nerve exclusively sensory ↓ Sensation in medial lower leg, medial ankle, foot arch

Question: What environmental accident is associated with tympanic membrane perforation?

Answer: Lightning strike. Rapid Review Tympanic Membrane Perforation Foreign body, infection, blast, ↑ barometric pressure Pars tensa most commonly perforated Pain, ↓ hearing, bleeding ​Keep ear dry, analgesics 90% heal in a few months

Question: What is a common treatment of papilledema due to idiopathic intracranial hypertension?

Answer: Lumbar puncture. Rapid Review Papilledema Due to ↑ ICP Pseudotumor cerebri Hypertensive emergency Tumors

Question: Chronic migraine is strongly associated with which disorder?

Answer: Medication overuse (rebound phenomenon or rebound headaches). Rapid Review Migraine Headache Gradual onset, unilateral > bilateral, throbbing, pulsating First episode: < 30 years old F > M Triggers: cheese, OCPs, pregnancy, menstruation Without aura: most common, N/V, photophobia, phonophobia Aura: scotoma, flashing lights, sounds Abortive rx: triptans, DHE, antiemetics, NSAIDs Ppx: ßBs. CCBs, TCAs Triptans, DHE: contraindicated in HTN or CV disease

Question: What acid-base disorder is seen in ethylene glycol poisoning?

Answer: Metabolic acidosis with positive anion gap.

Question: What is Miller Fisher syndrome?

Answer: Miller Fisher syndrome is a rare form of GBS characterized by the triad of ophthalmoplegia, ataxia, and areflexia. Rapid Review Guillain-Barré Syndrome Antecedent pulmonary or GI illness (Campylobacter jejuni) Rapid ascending symmetrical weakness Lower extremity weakness > upper extremity weakness Deep tendon reflex loss → respiratory failure Normal rectal tone CSF: markedly ↑ protein with up to 100 lymphocytes/μL Obtain pulmonary function tests Rx: IVIG or plasmapharesis, possible prophylactic intubation

Question: Which patients with altered mental status should receive empiric naloxone?

Answer: Naloxone is an opioid reversal agent and should be considered in patients who present with altered mental status with respiratory depression. Rapid Review Hypoglycemia Usually glucose <60 Confusion, agitation, unresponsiveness Tachycardia, diaphoresis, tremulousness Focal neurologic deficit Dextrose, thiamine, glucagon

Question: Is there a vaccine for lyme disease?

Answer: No. The manufacturer discontinued production of the vaccine due to poor sales and it is no longer available. Rapid Review Lyme Disease NE USA Campers/hikers Ixodes tick harbors Borrelia burgdorferi Stage I: erythema migrans (pathognomonic), viral-like syndrome Stage II: arthritis, myocarditis, bilateral Bell's palsy Stage III: chronic arthritis, chronic encephalopathy Rx: doxycycline CNS/cardiac involvement: IV ceftriaxone Pregnant/children rx: amoxicillin

Question: What role do neuromuscular blocking drugs have in refractory status epilepticus?

Answer: Nondepolarizing neuromuscular blocking drugs will stop the tonic-clonic movements, but will not affect abnormal neuronal firing. Patient will need an EEG. Rapid Review Status Epilepticus Seizure lasting > 5 minutes or > 2 discrete seizures without recovery of consciousness Causes: antiepileptic therapy discontinuation, medication noncompliance BZDs, phenytoin (second line), phenobarbital (third line) Make sure to check a blood sugar

Question: What autosomal dominant condition has epistaxis as its most common presenting symptom?

Answer: Osler-Weber-Rendu disease also called hereditary hemorrhagic telangiectasia (HHT) . Rapid Review Epistaxis Most common source: Anterior bleeds: Kiesselbach's plexus Posterior bleeds: Sphenopalatine artery Treatment is: Anterior bleeding: direct pressure, packing, cautery, Posterior bleeding: packing (foley, gauze pack, intranasal balloon device) Admit patients with posterior packing to a monitored bed

Question: What is the treatment of choice for primary syphilis infection?

Answer: Penicillin G. Rapid Review Bacterial Vaginosis: Gardnerella Vaginalis infection Clue cells on microscopic exam Amsel Criteria: must have 3/4 Treatment: metronidazole 500 mg, PO, BID, for 7 days

Question: What is a myasthenic crisis?

Answer: Respiratory failure leading to mechanical ventilation. Rapid Review Myasthenia Gravis Patient will be complaining of proximal muscle weakness, ptosis, and diplopia that is worse at the end of the day PE will show ice test improves sx Diagnosis is made by edrophonium (tensilon) test, EMG Most commonly caused by autoimmune destruction of acetylcholine receptors Treatment is acetylcholinesterase inhibitors, such as pyridostigmine Comments: associated with thymoma

Question: What bacteria most commonly cause peritonsillar abscesses?

Answer: S. pyogenes, S. aureus, and anaerobes.

Question: What are the most common organisms that cause orbital cellulitis?

Answer: Staphylococcus aureus and Streptococcus pneumoniae. Rapid Review Orbital Cellulitis S. aureus Painful/↓ EOM (unlike periorbital cellulitis) CT orbit IV ABX, ophthalmology consultation

Question: What is the most common bacteria responsible for infective endocarditis?

Answer: Staphylococcus aureus. Rapid Review Mitral Regurgitation Patient with a history of ischemic heart disease, endocarditis, MI, trauma Complaining of dyspnea PE will show blowing holosystolic mumur, best heard at apex with radiation to axilla, pulmonary edema, cardiogenic edema Diagnosis is made by echo Treatment is nitroprusside, dobutamine, intraaortic balloon pump, emergency surgery

Question: Which bacteria are most closely associated with dental caries?

Answer: Streptococcus mutans.

Question: The causative organism of Lyme disease, Borrelia burgdorferi, belongs to which class of organisms?

Answer: The organism that causes Lyme disease is a spirochete (a spiral-shaped bacteria) as is the organism that causes syphilis.

Question: All topical ocular medications with a red-colored cap have what property?

Answer: They are all mydriatics. Rapid Review Acute Angle-Closure Glaucoma Patient will be entering a dark room or movie theater Complaining of acute unilateral painful vision loss, vomiting, and seeing halos around lights PE will show cloudy cornea and fixed mid-dilated pupil Labs will show ↑ IOP ( > 21 mm Hg) Treatment is topical ßBs, carbonic anhydrase inhibitors, steroids, miotics

Question: Why are corticosteroids recommended in thyroid storm?

Answer: To inhibit conversion of T4 to T3, block the release of hormone from gland, and treat relative adrenal insufficiency. Rapid Review Hyperthyroidism Patient will be complaining of heat intolerance, palpitations, weight loss, tachycardia, and anxiety PE will show hyperreflexia Labs will show low TSH and high free T4 Most commonly caused by Graves disease (autoimmune against TSH receptor) Treatment is methimazole or PTU Comments: Propylthiouracil (PTU) P for pregnant

Question: What are some lifestyle modifications recommended for those with heart failure?

Answer: Tobacco cessation, limited alcohol and sodium intake, weight loss and regular exercise. Rapid Review Dilated Cardiomyopathy Causes: idiopathic > viral myocarditis, Chagas disease Echo: 4 dilated chambers (ventricles > atria) Systolic pump failure → heart failure S3, S4 Peripheral embolization of mural thrombi Most common dysrhythmia: atrial fibrillation ßBs, ACEIs, diuretics, digoxin, anticoagulation, AICD

Question: True or false: the majority of patients with primary biliary cirrhosis are women?

Answer: True.

Question: What formula is used to calculate the average weight in a child?

Answer: Weight (kg) = (2 x age in years) + 8. Rapid Review Lower Limits of Pediatric Systolic Blood Pressure 0-28 days: 60 mm Hg 1-12 months: 70 mm Hg 1-10 years: 70 mm Hg + (2 x age in years) >10 years: 90 mm Hg

Question: Will balancing out energy intake and expenditure correct menstruation irregularities?

Answer: Yes, with corrected energy balance as it relates to food intake and balanced exercise, menstruation can be normalized. Rapid Review Female Athlete Triad Patient will be a women With a history of intense athletic training Complaining of weight loss and irregular periods PE will show osteopenia Triad: Disordered eating, amenorrhea, osteopenia

Which vitamin deficiency presents with photosensitive dermatitis, diarrhea and dementia? Cyanocobalamin Niacin Pyridoxine Thiamine

Correct Answer ( B ) Explanation: Niacin (B3) deficiency causes the clinical presentation of pellagra - dermatitis, diarrhea and dementia. Niacin is a vitamin found in poultry, meat and fish. Deficiency is rare in developed countries but can be seen in patients with alcohol abuse or other chronically malnourished states. Patients will present with raw skin and a photosensitive-pigmented dermatitis that is more pronounced in sun-exposed areas. Additionally, diarrhea and dementia are commonly seen. Niacin administration is the focus of treatment although other concurrent vitamin deficiencies should be investigated. Cyanocobalamin (A) or vitamin B12 deficiency is typified by weakness, parasthesias and macrocytic anemia. Pyridoxine (C) or vitamin B6 deficiency presents with nausea and vomiting as well as fever and tachycardia. Intractable seizures may also be seen particularly with isoniazid toxicity. Thiamine (D) or vitamin B1 deficiency can present with dry beriberi (nervous system involvement including Wernicke's and Korsakoff syndromes) or wet beriberi (high output cardiac failure).

An 85-year-old man presents to the emergency department with altered mental status. Which finding would suggest a diagnosis of delirium rather than dementia? Acute onset Depressed mood Memory impairment Pupillary dilation

Correct Answer ( A ) Explanation: Delirium is a common mental disorder seen in patients with a medical illness, especially older adults. Delirium is characterized by an acute onset (usually hours to days) disturbance in attention and cognition that is caused by a medical condition, substance withdrawal or intoxication, or a side effect from medication. The changes in mental status cannot be explained by a preexisting condition and are not related to a reduction in level of arousal. Patients with delirium may also exhibit psychomotor behavioral and emotional disturbances. The differential diagnosis for delirium is extensive, and diagnostic evaluation involves both identifying that delirium is present as well as determining the underlying medical cause. Obtaining a history and performing a physical exam on a patient with delirium is challenging. History should be obtained from family members or friends and physical exam should include a focused assessment with review of vital signs, skin condition, level of hydration and clues to an infectious etiology. Treatment of delirium involves managing the behavior disturbance along with finding and treating the underlying medical cause. Elderly patients who present with depressed mood (B) or suicidal ideation may have pseudodementia which resolves with treatment of depression. A chronic, steady decline in short and long-term memory (C) is characteristic of dementia. Pupillary dilation (D) in a patient with altered mental status points to an etiology of substance abuse or intoxication, and may or may not be seen in patients with delirium.

A 60-year-old man presents with nighttime dyspnea. His medical history is significant for chronic hypertension. A recent echocardiogram showed an increase in left ventricular chamber volume but normal ventricular wall thickness. Based on this finding alone, which of the following medications is the most appropriate treatment for this patient's dyspnea? Bisoprolol Propranolol Salmeterol Timolol

Correct Answer ( A ) Explanation: Dilated cardiomyopathy (DCM) is a complication of chronic hypertension and coronary artery disease. Patients with cardiomyopathy typically suffer from symptoms of cardiac failure, especially dyspnea and edema. If cardiac failure is suspected, the initial evaluation for underlying cardiomyopathy includes an echocardiogram. In DCM, the echocardiogram usually demonstrates an enlarged ventricular chamber with normal or decreased wall thickness. Ejection fraction, a marker of systolic function, will also typically be normal or decreased. Treatment of DCM follows the American College of Cardiology and the American Heart Association's guidelines for the treatment of heart failure. In addition to lifestyle modifications, pharmacotherapy includes ACE-inhibitors or ARBs, loop diuretics and beta-blockers. Bisoprolol is a selective beta-1 adrenergic receptor blocker. Bisoprolol, carvedilol, and metoprolol, but not propranolol (B), are the only beta-blockers with proven benefit in heart failure management. Salmeterol (C) is a long-acting beta-2-agonist used in treating bronchospasm and COPD, not heart failure. Although this medication could be used for treating dyspnea in a patient with pulmonary disease, there is no information about this patient's pulmonary status given in the question stem above. Timolol (D) is a beta-blocker used in the treatment of glaucoma, not heart failure.

A 25-year-old woman visiting from India presents to the Emergency Department with a complaint of a painless ulcer on her vulva. She states it started as a nodule a week prior, but then erupted as a large red ulcer that bleeds easily, even after light contact. She denies fevers, chills, dysuria, or vaginal discharge. On examination, a "beefy-red" ulcer is noted on her vulva. No lymphadenopathy is appreciated. No other vesicles, lesions, or rashes are visualized. Which of the following is the most likely diagnosis? Donovanosis Herpes simplex virus Lymphogranuloma venereum Syphilis

Correct Answer ( A ) Explanation: Donovanosis, or granuloma inguinale, is caused by Klebsiella granulomatis, a gram negative intracellular bacterium. The disease is most prevalent in India, southern Africa, and central Australia. It is rare in the United States. The incubation period lasts for 2 weeks to 6 months, after which time a painless subcutaneous nodule forms on the penis or vulva. The nodule progresses to a painless ulcer with rolled borders that is highly vascular, giving it the classic "beefy-red" appearance. Due to the vascularity, the lesions bleed easily with contact. Lymphadenopathy is not present. Diagnosis is made by visualization of Donovan bodies (safety-pin shaped intracellular organisms) in monocytes on tissue biopsy. Treatment is with doxycycline for at least 3 weeks. Alternative medications include azithromycin, ciprofloxacin, erythromycin, and trimethoprim-sulfamethoxazole. Herpes simplex virus (B), type 1 or 2, initially presents as grouped small painful vesicles after a viral prodrome. Typically the patient will also complain of localized tingling, burning, and pain prior at the site where the vesicles eventually develop. These vesicles eventually deroof and become painful shallow ulcers. Tender lymphadenopathy may develop, but does not progress to large fluctuant nodes and will not develop above and below the inguinal ligament. Lymphogranuloma venereum (C) (LGV) is a sexually transmitted disease more prevalent in tropical countries and is typically more prevalent in the homosexual population. It is caused by Chlamydia trachomatis, a gram negative intracellular bacteria. Incubation lasts for 3 to 21 days when a small, painless genital ulcer appears and spontaneously resolves after 2 to 3 days. The secondary stage begins 7 to 30 days after the resolution of the primary lesion and is characterized by buboes which are painful, firm fluctuant lymph nodes which form a "groove sign" (adenopathy that extends above and below the iinguinal ligament). Syphilis (D), caused by the spirochete Treponema pallidum, first presents as a chancre or painless ulcer with indurated borders lasting 3 weeks. This ulcer does not start out as a subcutaneous nodule and the ulcer is not a "beefy-red" color. THe chancre typically heals within 3 to 6 weeks, however, secondary (characterized by a rash and lymphadenopathy) and tertiary syphilis can ensue and affect the nervous and cardiovascular system through neuropathy and widespread granulomatous lesions.

A 14-year-old boy presents with left seventh nerve facial palsy and a rash covering most of his body, diagnosed by a dermatologist as erythema migrans. He is also complaining of a headache, mild neck stiffness, myalgia, low-grade fever, malaise, and arthralgia. A large tick is removed from his scalp. Borrelia burgdorferi infection is diagnosed. Select the one finding below that is most commonly and specifically associated with Lyme disease? Erythema migrans History of a large tick bite Low-grade fever Meningitis

Correct Answer ( A ) Explanation: Erythema migrans, the characteristic skin rash of Lyme disease, occurs in two-thirds of patients with Lyme disease. The erythema expands over days, slowly clearing in the center, forming a target-like lesion around the original tick bite. As the Borrelia burgdorferi organisms disseminate, multiple lesions are formed all over the body. The history of a tick bite (B) is elicited in only 25 percent of patients diagnosed with Lyme disease. The Ixodes tick vector for this disease is very small (2 mm) and difficult to see. Low-grade fever (C) is seen in less than half of patients with Lyme disease and is very non-specific for Lyme disease. Meningitis (D) and facial nerve palsy are manifest in less than one-fifth of Lyme disease cases. The meningitis is described as waxing and waning, associated with mild to moderate neck stiffness. Cranial neuropathies including Bell's palsy, develop in 3 percent of Lyme disease patients. However, in Lyme endemic areas during the tick season, a significant percentage of individuals diagnosed with Bell's palsy, have Lyme disease.

A 6-year-old boy presents with a fever of 38.5°C, sore throat, and tender anterior cervical adenopathy. He does not have a cough or a runny nose. His younger sister was treated for streptococcal pharyngitis last week and his mother would like him to be treated for streptococcal infection. According to the modified Centor criteria, which of the following is the most appropriate action? Empiric antibiotic treatment for streptococcal pharyngitis Rapid antigen detection testing Streptococcal antibody titers Tonsillectomy when he recovers from this infection

Correct Answer ( A ) Explanation: Pharyngitis is diagnosed in 11 million patients in U.S. emergency departments and ambulatory settings annually. Most episodes are viral, however Group A beta-hemolytic Streptococcus is the most common bacterial etiology. Common signs and symptoms of streptococcal pharyngitis include sore throat, fever, tonsillar exudates, and cervical adenopathy. Cough, coryza, and diarrhea are more common with viral pharyngitis. Available diagnostic tests include throat culture and rapid antigen detection testing. Testing and treatment is based on the Centor scoring system, which estimates the probability of streptococcal pharyngitis and the need for antibiotics. The 5 criteria for the Centor scoring system include: the presence of fever, tonsillar exudate, anterior cervical adenopathy and absence of cough in patients 3-14 years of age. Patients with a score of zero or 1 are at very low risk for streptococcal pharyngitis and do not require testing (throat culture or rapid antigen detection testing) or antibiotic therapy. Patients with a score of 2 or 3 should be tested using rapid antigen detection test or throat culture; positive results warrant antibiotic therapy. Patients with a score of 4 or higher are at high risk of streptococcal pharyngitis, and empiric treatment may be considered without the need for testing. This patient has a modified Centor criteria score of 4 and therefore should be treated for streptococcal pharyngitis. Penicillin (10 days of oral therapy or one injection of intramuscular benzathine penicillin) is the treatment of choice because of cost, narrow spectrum of activity, and effectiveness. Amoxicillin is equally effective and more palatable. Erythromycin and first-generation cephalosporins are options in patients with penicillin allergy. Rapid antigen detection testing (B) and throat culture are used for patients with suspected Group A beta-hemolytic Streptococcus with a Centor score of 2-3. This patient however has 4 of the 5 clinical criteria for Group A beta-hemolytic Streptococcus and therefore needs treatment with antibiotics. Streptococcal antibody titers (C) are not useful for diagnosing streptococcal pharyngitis and are not routinely recommended. They may be indicated to confirm previous infection in persons with suspected acute poststreptococcal glomerulonephritis or rheumatic fever. They may also help distinguish acute infection from chronic carrier status, although they are not routinely recommended for this purpose. The effect of tonsillectomy (D) on decreasing risk for chronic or recurrent throat infection is poorly understood and there is not conclusive evidence that supports the benefits of a tonsillectomy.

A 34-year-old man presents with complaints of progressive lower extremity weakness over the past 2 days associated with 2 weeks of diarrhea that has since resolved. Vital signs are HR 89, BP 160/95, and RR 12. On exam, you note symmetric lower extremity weakness with intact sensation and absent ankle reflexes. Which of the following would help to support the most likely diagnosis? Albumino-cytological dissociation Improvement with steroids Increased forced vital capacity Selective enhancement of the dorsal root ganglion on MRI

Correct Answer ( A ) Explanation: The patient is most likely suffering from Guillain-Barré syndrome, a progressive, ascending demyelinating polyneuropathy that typically begins with lower extremity weakness. Classically, GBS presents with symmetric lower extremity weakness, decreased or absent deep tendon reflexes, and little or no sensory involvement (although variable). It follows a respiratory or gastrointestinal infection by weeks to days. Common organisms implicated include Campylobacter jejuni, cytomegalovirus, Epstein-Barr virus, and Mycoplasma pneumoniae. Elevated protein with only a mild pleocytosis on cerebrospinal fluid analysis (termed albumin-cytological dissociation) is highly specific in the clinical setting of suspected GBS. Patients with probable GBS should receive a neurology consult and be admitted for either intravenous immunoglobulin (IVIG) therapy or plasma exchange. Both IVIG and plasma exchange have been demonstrated to be more effective than placebo has, but no advantage is incurred upon administration of both therapies. Neither therapy has been proven more effective than the other. Selective enhancement of the anterior spinal nerve roots (D) on MRI is suggestive. Nearly 33% of patients diagnosed with GBS will require ventilator support. Factors associated with an increased need for ventilator support include a decreased forced vital capacity (FVC) (C) and a negative inspiratory force (NIF) less than 30 cm water. Other useful adjuncts include peak expiratory flow rate (PEFR) and arterial blood gas to evaluate for carbon dioxide retention. Corticosteroids alone (B) are no longer recommended, but IV steroids coadministered with IVIG may speed recovery. Regardless, steroids have no role in the diagnosis of GBS.

A 46-year-old man with a history of depression presents with a self inflicted laceration to the left arm. The wound is superficial and the patient states that he didn't want to hurt himself. He reports that he is recently divorced, lives alone, and has increased stress at work. These stressors have worsened his depression. Which of the following treatments is most likely indicated for this patient? Emergency psychiatric evaluation Outpatient referral to psychiatry Prescribe antidepressant and discharge with follow up Prescribe anxiolytic and discharge

Correct Answer ( A ) Explanation: This patient presents with a possible suicide attempt and a number of high risk factors for suicidality warranting an emergency evaluation from psychiatry. Patients often present to the emergency department after an attempt that may appear minor. Many of these patients have a normal mental status making it vital to obtain details about the occurrence as well as the context. Additionally, patients may experience shame about the event and attempt to explain it as an accident. It is central in management for the physician to treat all possible suicide attempts as the serious events they are. All of these patients have the potential to attempt again and may be successful. For this reason, any patient presenting with a possible suicide attempt should have an emergent psychiatric evaluation. Although anxiolysis (D) may be necessary in the ED to keep the patient calm, discharging them on an anxiolytic without a psychiatric evaluation is dangerous. Likewise, prescribing an antidepressant (C) without further evaluation can be harmful. Both of these medications may be used in a future attempt. After psychiatric evaluation, the patient may be deemed to be safe for follow up as an outpatient (B) but this determination should not be made until a full psychiatric assessment is performed.

Which of the following represents appropriate management of a thrombosed external hemorrhoid? Excision Reduction Sitz baths Surgery referral

Correct Answer ( A ) Explanation: Thrombosed external hemorrhoids presenting within 48 - 72 hours of symptom onset should be excised. Hemorrhoids occur when small vascular beds become engorged. Both external and internal hemorrhoids can present with the complaint of bright red blood per rectum. External hemorrhoids, though, are typically painful while internal ones are not. Nonthrombosed external hemorrhoids can be managed with warm water baths, analgesics, stool softeners, and high-fiber diet. Acutely thrombosed external hemorrhoids, however, will not improve with conservative therapy and require excision. Incision of the hemorrhoid is followed by complete evacuation of the clot. Failure to fully remove the clot results in rebleeding, swelling and skin tag formation. If the thrombosed external hemorrhoid is not excised, it will go on to ulcerate over the next several days. Reduction (B) should be performed on nonthrombosed external hemorrhoids if possible. Sitz baths (C) are helpful for nonthrombosed external hemorrhoids. Surgical referral (D) should not replace excision.

A 63-year-old man presents to your office complaining of the gradual onset over the last year of nocturia, urinary frequency, and change in urinary stream. He otherwise feels well. His physical exam is notable for a uniformly enlarged, non-tender prostate gland. Which of the following is the most likely diagnosis? Acute bacterial prostatitis Benign prostatic hyperplasia Urinary tract infection Vesicoureteral reflux

Correct Answer ( B ) Explanation: Benign prostatic hyperplasia (BPH) is common as men age and can cause partial urethral obstruction that is characterized by incomplete voiding, urinary frequency, straining to begin urination, urgency, nocturia, and a change in force of the urinary stream due to enlargement of the prostate gland. Occasionally, renal function is impaired. On exam, the prostate gland will feel smooth and symmetrically enlarged and the bladder may be distended. The diagnosis is mainly clinical. Postvoid residual urine volume and urodynamic studies can be used to indicate the degree of obstruction. Prostate specific antigen (PSA) may be elevated but is nonspecific. Serum creatinine and other signs of renal impairment may be elevated due to obstruction and resultant hydronephrosis. Treatment is primarily symptomatic and is aimed at preventing or treating clinically significant urinary obstruction. Alpha-blockers (doxazosin, terazosin, prazosin, tamsulosin) relieve symptoms by relaxing smooth muscles of the bladder neck. 5-alpha-reductase inhibitors (finasteride, dutasteride) shrink the prostate, but do not acutely relieve symptoms. Surgical management involves transuretheral resection of the prostate (TURP). Unless secondary complications are present, patients with BPH do not present with symptoms of acute illness such as fevers, chills, rectal pain or malaise. Patients with acute bacterial prostatitis (A) present with perineal, rectal and sometimes lower back pain, fevers, chills and malaise. Patients may develop urinary retention, and on digital exam, the prostate is usually very tender. Urinary tract infection (C) is characterized by irritative voiding symptoms such as dysuria, urgency and incontinence, though patients may present with fevers, chills, nausea, vomiting and flank pain if they have developed pyelonephritis. Vesicoureteral reflux (D) happens when urine flows from the bladder to the upper urinary tract and is the most likely mechanism for the development of pyelonephritis in children.

What is the most common symptom seen in acute bronchitis? Chest pain Cough Fever Sputum

Correct Answer ( B ) Explanation: Bronchitis is defined as inflammation of the lower airways and is typically caused by viral pathogens. The most common presenting complaint is cough in these patients. In healthy adult patients, acute bronchitis is typically caused by a viral pathogen. As such, it is not amenable to antibiotic therapy. In patients with chronic bronchitis or chronic obstructive pulmonary disease (COPD), exacerbations of symptoms may be the result of bacterial pathogens causing acute bronchitis on top of their chronic bronchitis. Treatment with antibiotics should be considered in this subset of patients with bronchitis. Chest pain (A) often occurs as a result of coughing in patients with bronchitis. Fever (C) may or may not be present and does not indicate a higher likelihood of bacterial infection. Sputum (D) is present in about half of patients with acute bronchitis.

A 18-year-old man presents with a depressed mental status. Which of the following could be empirically given? Activated charcoal Dextrose and thiamine Flumazenil Syrup of ipecac

Correct Answer ( B ) Explanation: In the undifferentiated patient with depressed mental status, empiric delivery of dextrose and thiamine should be considered. Hypoglycemia can manifest with varied presentations. Patients may exhibit lethargy, somnolence, agitation, confusion, altered mental status, seizures, focal neurologic deficits or coma. In all patients with neurologic changes, a rapid finger stick should be obtained. However, in the absence of a finger stick, it is reasonable to empirically treat with dextrose as hypoglycemia is life threatening. 1-2 grams of dextrose/kg should be given intravenously. In patients in whom intravenous access cannot be established, glucagon can be given intramuscularly. This will increase circulating glucose levels by mobilizing glycogen stores. However, it will not work in those patients with limited glycogen stores (chronically ill, malnourished etc.). Thiamine should be given with dextrose to prevent the development of Wernicke's encephalopathy although the traditional "thiamine before dextrose" has little basis in the literature. Activated charcoal (A) should not be given to patients with depressed mental status if their airway is not protected as aspiration of activated charcoal is extremely hazardous. Flumazenil (C) is a benzodiazepine antagonist. It should not be given empirically as it removes the ability to control seizures in patients taking first line medications. Syrup of ipecac (D) induces vomiting and can lead to aspiration in a patient not protecting their airway.

A 44-year-old woman presents with a bilateral throbbing headache and nausea that was preceded by vision changes of flashing lights and blind spots. Her past medical history is significant for poorly controlled hypertension. Currently, her blood pressure is 182/100 and neurological examination is normal. CT scanning reveals no intracranial pathology. Which of the following is the most appropriate headache treatment at this time? High flow oxygen Ibuprofen Sumatriptan alone Sumatriptan plus ergotamine

Correct Answer ( B ) Explanation: Migraine headaches are characterized by 4-72 hours of unilateral or bilateral head pain which is associated with neurologic, autonomic and psychological symptoms. Three subtypes exist. Migraine with aura only accounts for 20% of migraines. This type of headache is preceded by 30 minutes to 1 hour of prodrome symptoms: weakness, paresthesias and several visual changes including scotoma (blind spots), hemianopia, fortification (zizag patterns) and scintilla (flashing lights). Migraine without aura is headache without preceding prodrome symptoms. Finally, chronic migraine (or episodic migraine without aura) is a progressive form characterized by increasing symptom frequency and comorbid psychiatric conditions such as depression, anxiety, bipolar disorder, panic disorder and obsessive-compulsive disorder. The pathophysiological basis of migraine is brain "hypersensitivity" caused by trigeminal nerve and upper cervical nerve excitation of perivascular inflammation. Up to 90% of migraine sufferers have a positive family history. The majority of patients are adult females. Associated nonprodromal symptoms include nausea and vomiting, sensitivity to light (photophobia; prefer to be left alone in a dark room), sensitivity to sound (phonophobia), blurred vision and polyuria. Abortive treatment consists of acetaminophen-aspirin-caffeine, NSAIDs (ibuprofen), antiemetics, ergotamine and triptan medications. Preventive treatment of migraine consists of tricyclic antidepressants, beta-blockers, anticonvulsants, ergot-derivatives, tizanidine, occipital nerve blocks, botulinum toxin injection and calcium channel blockers. High flow oxygen (A) is used in treating acute cluster headache and not migraine. This scenario does not match the description of cluster headaches. Triptan medications (C and D) have adrenergic-agonistic properties. Therefore, they should not be used in patients with cardiovascular or cerebrovascular disease, angina, severe hypertension and peripheral vascular disease. Clinicians must never combine ergot-derivatives with triptan medications.

A 49-year-old man presents with chest pain. His medical history does not list any cardiac murmur, however, during examination, you hear a mitral regurgitant murmur. Blood pressure is equal in both the left and right arms. Although you have none to compare to, you order an electrocardiogram and notice ST segment depression in three different leads and T-wave inversion in two different leads. No other abnormalities are appreciated. A chest radiograph is read as normal. Initial lab testing shows an elevated troponin level. Which of the following is the most correct diagnosis? Aortic dissection Non ST-segment elevation myocardial infarction ST-segment elevation myocardial infarction Unstable angina

Correct Answer ( B ) Explanation: Myocardial infarction encompasses both non-ST-segment elevation myocardial infarction (NSTEMI) and ST-segment elevation myocardial infarction (STEMI). It is defined as myocardial cell death and necrosis as diagnosed by a rise and fall in cardiac enzymes (in association with appropriate clinical presentation) or by pathologic findings of prior myocardial infarction (e.g. new Q waves on ECG). NSTEMI represents subtotal coronary thrombosis and myocardial ischemia infarct. Common symptoms include angina less than 30 minutes duration, dyspnea, diaphoresis and palpitations. Myocardial insult is also associated with a new-onset mitral regurgitation murmur, newly-auscultated S4 and a paradoxical S2. New infarcts can be identified with a positive troponin on laboratory testing. Aortic dissection (A) is associated with severe, tearing, knifelike pain that may radiate to the midscapular region. It is associated with asymmetric blood pressures in the arms, aortic regurgitation murmur, widened mediastinum on radiography and neurologic deficits. It is less likely than NSTEM in this scenario. STEMI (C) is associated with positive troponins and ST-segment elevation, not depression. Unstable angina (D) is not associated with positive troponin levels, but it can be associated with T wave inversion and ST depression.

A 23-year-old woman presents to your office with complaints of pelvic pain and vaginal discharge. She admits to having recent unprotected intercourse with a new partner and her last menstrual period ended three days ago. Which of the following physical exam findings supports the likely diagnosis? Chadwick sign Chandelier sign Goodell's sign Hegar's sign

Correct Answer ( B ) Explanation: Pelvic inflammatory disease (PID) is an acute infectious condition of the upper female genital tract that causes significant inflammation. The most common areas of infection include the uterus and fallopian tubes, although adjacent pelvic structures or areas in the abdomen may also be affected. Risk factors for PID include age younger than 25 years, multiple sexual partners, unprotected intercourse, and a past history of sexually transmitted infections (STIs). Neisseria gonorrheae and Chlamydia trachomatis are the pathogens most commonly identified in patients with PID. The most common presenting symptom is lower abdominal pain. Patients may also complain of abnormal bleeding, vaginal discharge, fever, chills, nausea, or vomiting, depending on the severity of disease. Physical exam findings include cervical motion tenderness, also referred to as chandelier sign. Cervical or adnexal tenderness on bimanual exam is suggestive of PID and further workup including testing for the presence of Neisseria gonorrhea and Chlamydia trachomatis should be initiated. Chadwick sign (A), Goodell's sign (C) and Hegar's sign (D) are all physical exam findings suggestive of pregnancy. Chadwick sign is cyanosis of the cervix due to venous congestion seen at 8-10 weeks of pregnancy. Goodell's sign is a softening of the cervix present at 6 weeks of pregnancy. Hegar's sign is a softening of the uterus seen at 6-8 weeks of pregnancy.

In the patient with an enlarged prostate, which of the following drugs causes involution of benign prostatic tissue and prostate shrinkage? Doxazosin (Cardura®) Finasteride (Proscar®) Tadalafil (Cialis®) Vardenafil (Levitra®)

Correct Answer ( B ) Explanation: The 5 alpha-reductase inhibitors have been available since the early 1990s and finasteride was the first agent in this class. Finasteride prevents the conversion of testosterone to the more active metabolite dihydrotestosterone in the prostate. This inhibition results in involution of BPH tissue and prostate shrinkage. On average, most men achieve 20% to 40% reduction in prostate size after at least 6 months of use. In general, these agents are most effective in men with prostate glands more than 30 g. Urologists also use 5 alpha-reductase inhibitors to treat chronic hematuria due to an enlarged prostate and sometimes prescribe these agents before transurethral resection of the prostate (TURP) to lessen surgical bleeding. Finasteride is also indicated for hair-regrowth of the scalp. Doxazosin (A) is an alpha-adrenergic blocking oral agents to treat BPH has been commonplace since the 1980s. These agents are directed at the dynamic component of BPH and LUTS by relaxing the smooth muscle tissue in the bladder neck and prostate. In simple terms, they relax the bladder outlet, resulting in better urinary flow. Both tadalafil (C) and vardenafil (D) are phosphodiesterase type 5 inhibitors that work by relaxing smooth muscle and allowing the penis to fill with blood.

A 26-year-old man was involved in an automobile collision and sustained a pelvic fracture. He complains of medial ankle paresthesias which radiate into the arch of his foot. Which of the following nerves is most likely involved? Obturator Saphenous Superficial peroneal Sural

Correct Answer ( B ) Explanation: The largest nerve to emerge from the lumbosacral plexus is the femoral nerve, whose distal branches innervate the anterior thigh muscles and skin. Its most distal branch, the saphenous nerve, is a pure sensory nerve that transmits sensation from the medial lower leg, medial ankle and arch of the foot. The main contribution to the saphenous nerve is from the L4 root. The lumbosacral plexus lies anterior to the sacroiliac joints within the pelvis, and can be damaged post pelvic trauma, especially from an anterior-to-posterior force which results in sacroiliac joint disruption. A proximal femoral nerve injury may result in distal saphenous symptoms. The obturator nerve (A) is another branch of the lumbosacral plexus. It innervates the thigh adductors and only has a small sensory contribution to the medial thigh, superior to the knee. The superficial peroneal nerve (C) sends sensory branches to the skin of the anterolateral lower leg and dorsum of the foot. The sural nerve (D) is also a distal lower extremity sensory nerve, but it supplies sensory innervation to the posterolateral lower leg, lateral ankle and lateral foot.

A 53-year-old man presents with numbness to his right hand for 3-4 months. He states that he has numbness with waking up in the morning, which gets better when he shakes his hands. The patient indicates numbness to his first, second, and third digits on the right hand. Which of the following tests is most sensitive for this diagnosis? Finkelstein's test Median nerve compression test Phalen's test Tinel's sign

Correct Answer ( B ) Explanation: The patient presents with signs and symptoms consistent with carpal tunnel syndrome (CTS). CTS is the most common entrapment neuropathy of the wrist and is caused by compression of the median nerve. Associated symptoms include numbness and weakness in the distribution of the median nerve. Patients typically present with numbness, pain and parasthesias of the first three digits that are worse at night and with activity but improve with "shaking their hands out." Median nerve compression testing (Durkan's test) consists of direct pressure application to the median nerve at the carpal tunnel. It is positive if this maneuver reproduces the symptoms and is 87% sensitive and 90% specific for the diagnosis. Conservative treatment with wrist splinting and cortisone injections often relieves symptoms. Referral for orthopedic management is indicated in patients with recurrent or refractory symptoms. Finkelstein's test (A) involves forced ulnar deviation of the wrist with the thumb adducted and is diagnostic of de Quervain's tenosynovitis. Phalen's (C) and Tinel's (D) test are classically taught in the diagnosis of CTS but suffer from low sensitivity (76% and 42-85% respectively).

A 73-year-old man with benign prostatic hyperplasia presents with a failure to void for 3 days and lower abdominal pain. Physical examination reveals lower abdominal distension and moderate tenderness to palpation. His BUN/Creatinine is 8/1.6. What management is indicated? Emergency hemodialysis Foley catheter placement Intravenous fluids Suprapubic catheterization

Correct Answer ( B ) Explanation: The patient presents with symptoms and signs consistent with acute urinary obstruction requiring placement of a Foley catheter to relieve the obstruction. Postrenal or obstructive renal failure is a reversible cause of acute kidney injury (AKI) and should always be considered in patients with new azotemia. Benign prostatic hyperplasia (BPH) is a common cause of urinary obstruction. Treatment of obstructive uropathy focuses on relieving the obstruction. There is no contraindication to placing a Foley catheter in patients with BPH and it should be done emergently to relieve obstruction. Emergency dialysis (A) should only be undertaken for pulmonary edema, severe uncontrollable hypertension, hyperkalemia, severe electrolyte or acid-base disturbance, specific overdoses and severe, symptomatic uremia. Intravenous fluids (C) to restore circulating volume are not necessary in obstructive renal failure. Suprapubic catheterization (D) should be attempted if placement of a Foley catheter fails.

A 20-year-old sexually active woman presents with dysuria and polyuria for a one-week duration. She has never had a urinary tract infection. She denies hematuria, flank pain, suprapubic pain, or fever. She denies itching or vaginal discharge. A urine specimen taken earlier in the week showed significant pyuria but a culture resulted in no growth. She has taken an antibiotic for 2 days without relief. Her only other medication is an oral contraceptive agent. Which one of the following is the most likely infectious agent? Candida albicans Chlamydia trachomatis Escherichia coli Staphylococcus saprophyticus

Correct Answer ( B ) Explanation: Women who present with symptoms of acute dysuria, frequency, and pyuria do not always have bacterial cystitis. In fact, up to 30% will show either no growth or insignificant bacterial growth on a midstream urine culture. Most commonly these patients represent cases of sexually transmitted urethritis caused by Chlamydia trachomatis, Neisseria gonorrhoeae, or Herpes simplex virus. In this case, the gradual onset, absence of hematuria, and week long duration of symptoms suggest a sexually transmitted disease. A history of a new sexual partner or a finding of mucopurulent cervicitis confirms the diagnosis. Empiric treatment with doxycycline and ceftriaxone, as well as a search for other sexually transmitted diseases is also indicated. Another possible diagnosis is urinary tract infection with Escherichia coli or Staphylococcus species; however, the onset of these infections is usually abrupt and accompanied by other signs, such as suprapubic pain, hematuria, urgency, and frequency. Escherichia coli. (C) is the most common pathogen found in urinary tract infections and will result in a positive culture. Staphylococcus saprophyticus (D) causes 10-20% of urinary tract infections in females 17-27 years of age and is the second most common cause of urinary tract infections, after Escherichia coli. Sexual activity increases the risk of S. saprophyticus because bacteria are displaced from the normal flora of the vagina and perineum into the urethra. Most cases occur within 24 hours of sex, earning this infection the nickname "honeymoon cystitis." A positive culture urine culture confirms the diagnosis. Candida albicans (A) is unlikely because there is no accompanying whitish discharge or itching, and the patient's symptoms predate the use of antibiotics. Fungal infections often cause a thick "cottage cheese" like discharge and can be potentiated by the use of antibiotics.

Which of the following patients is considered hypotensive? A 1-year-old boy with a systolic blood pressure of 75 mm Hg A 2-year-old girl with a systolic blood pressure of 80 mm Hg A 3-year-old girl with a systolic blood pressure of 70 mm Hg A 6-year-old boy with a systolic blood pressure of 85 mm Hg

Correct Answer ( C ) Explanation: A 3-year-old girl with a systolic blood pressure of 70 mm Hg is considered to be hypotensive. The minimum systolic blood pressure in a child (1-10 years) is calculated by the formula: Minimum SBP = 70 + (2 x age in years). Therefore, the minimum systolic blood pressure for a 3-year-old is 76 mm Hg. The minimum systolic blood pressure for a neonate (0-28 days) is 60 mm Hg and an infant (1-12 months) is 70 mm Hg. Based on the formula, a systolic blood pressure of 75 mm Hg is considered normal for a 1-year-old (A). A systolic blood pressure of 80 mm Hg is normal for 2-year-old (B) and a systolic blood pressure of 85 mm Hg for a 6-year-old (D).

A 54-year-old woman has pain and swelling of the right knee. Examination of the synovial fluid reveals a leukocyte count of 5,000, and rhomboid-shaped crystals. Which of the following is the most likely diagnosis? Gonococcal arthritis Gout Pseudogout Rheumatoid arthritis

Correct Answer ( C ) Explanation: Acute monoarthritis can be the initial manifestation of many joint disorders. The first step in diagnosis is to verify that the source of pain is the joint, not the surrounding soft tissues. A careful history and physical examination are important because diagnostic studies frequently are only supportive. Examination of joint fluid often is essential in making a definitive diagnosis. The synovial fluids should be examined under polarized microscopy. Microscopic examination of synovial fluid in a patient suffering an acute attack of pseudogout shows large numbers of polymorphonuclear leukocytes. Calcium pyrophosphate dihydrate crystals are frequently found extracellularly and in polymorphonuclear leukocytes. When viewed with polarized light, the crystals appear as short, blunt rods, rhomboids, and cuboids. Plain radiographs of the affected joint will show cartilage calcification in the affected joint. Calcium pyrophosphate deposition is associated with inflammation of the knees, wrists, shoulders, ankles, elbows and or hands. There is no definitive treatment, however intra-articular corticosteroids, NSAIDs and colchicine can be effective for both the acute and chronic forms. Patients with disseminated Neisseria gonorrhoeae infection typically display a migratory pattern of arthralgias, tenosynovial inflammation, or nonerosive arthritis. A synovial fluid analysis reveals the presence of white blood cells in the absence of crystal deposits. Synovial cultures are variable, with a positive result in only 25 to 70 percent of patients with gonococcal arthritis (A). Gout (B) is characterized by painful joint inflammation, most commonly in the first metatarsophalangeal joint, resulting from precipitation of urate crystals in a joint space. The cultures are sterile in gout and "rat-bite" erosions may be seen on joint radiographs. Diagnosis is confirmed by identification of negatively birefringent, needle-like monosodium urate crystals in synovial fluid aspiration. Rheumatoid arthritis is characterized by symmetric, inflammatory joint pain that is worse in the morning and is associated with warmth, erythema and swelling of the joints. Synovial fluid analyses is not needed to diagnose rheumatoid arthritis (D), instead it is based on a timeline of clinical signs and symptoms as well as radiographic changes and serum rheumatoid factors.

Using the 2005 American Diabetes Association (ADA) glycemic control goals, which of the following scenarios would prompt a change in insulin dosing in a patient with type 1 diabetes mellitus? A 10-year-old boy whose fasting serum glucose is 130 mg/dL A 14-year-old girl whose hemoglobin-A1c is 6.8% A 4-year-old girl whose hemoglobin-A1c is 7.8% An 8-year-old boy whose hemoglobin-A1c is 5.9%

Correct Answer ( C ) Explanation: Both forms of diabetes are properly treated with a multidisciplinary approach. Diet modification and education are necessary (dietician). Disease education is helpful (primary or midlevel provider). Injection procedure and glucose monitoring education is also important (primary or midlevel provider). Exercise modification may also be needed (physical therapist). There are several different classes of medications used in the treatment of diabetes. A special case is the child with type 1 diabetes mellitus. This child will likely be on insulin therapy for life. Therefore, monitoring and regimen modification becomes a large responsibility of primary care providers. The ADA recommends hemoglobin-A1c testing for monitoring and insulin adjustment. In 2014, the ADA released a position statement on the diagnosis and management of type 1 diabetes in all age groups. The statement includes a new pediatric glycemic control target of HbA1c of less than 7.5% across all pediatric age groups, replacing earlier guidelines that specified different glycemic control targets by age. The adult HbA1c target of less than 7% did not change. The 4-year-old girl whose hemoglobin-A1c is 7.8% needs a change (increase) in her insulin dosing as her hemoglobin-A1c is too high and the target A1c level is < 7.5%. A 14-year-old girl whose hemoglobin-A1c is 6.8% (B) and 8-year-old boy whose hemoglobin-A1c is 5.9% (D) are at goal. As such, a change in insulin is not necessary. Although a 10-year-old boy whose fasting serum glucose is 130 mg/dL (A) has DM1 based on the question stem, his abnormally high fasting serum glucose would not trigger an automatic re-titration of insulin because it is not the measure used to adjust insulin dosing in a child with DM1. The correct measure is the hemoglobin-A1c, not the fasting glucose.

A 6-year-old girl presents to the clinic for a well child visit. She has seen a dentist in the past, but her last visit was over one year ago. On physical exam, she is noted to have white spot lesions on the proximal surfaces of her incisors. There are no areas of cavitation in the teeth. Which of the following is the most appropriate information to give this family? Caries are more common in higher socioeconomic classes due to increased sugar consumption Early childhood caries is an inherited condition not affected by diet Fluoride supplementation is the most effective preventive measure against dental caries Nearly 25 percent of US children have dental caries

Correct Answer ( C ) Explanation: Dental caries develop as a result of interactions between the tooth surface, oral bacteria, and dietary carbohydrates. As the oral bacteria ferment carbohydrates, organic acids are produced and the oral pH is decreased. This leads to demineralization of the tooth surface, which is seen as a "white spot lesion" on the enamel. As demineralization continues, cavitation of the tooth will develop. There are many factors that influence demineralization and increase the risk of caries. The type of carbohydrate consumed can influence demineralization: sucrose is the most cariogenic polymer. In addition, the duration that sugar is in the mouth also affects the risk of caries; sticky candies and bottles with sweetened beverages increase demineralization. Factors that decrease the risk of dental caries include decreasing sugary beverages in the diet, good dental hygiene, and a dental home. In addition, fluoride supplementation in communal water (1 ppm fluoride) is the best preventive measure against caries. Topical fluoride in the form of toothpaste can reduce caries. Fluoride varnish administered by professionals to children at risk for caries can reduce caries by up to 30%. Dental caries are more commonly found in low socioeconomic groups (A), as well as children in developing countries. Early childhood caries (B) is a condition characterized by frequent dental caries in infants and toddlers. It is caused by cariogenic bacteria, frequent consumption of sugar, and is increased in certain populations such as low socioeconomic status, recent immigrants, and Native American groups. More than 50% of US children have dental caries (D). Establishing a dental home for all children is important in the prevention and treatment of dental caries.

You suspect a 35-year-old man has epiglottitis with impending airway compromise. Which of the following is the most accurate method for confirming the diagnosis? Computed tomography of the neck Frontal cervical soft tissue radiograph Indirect laryngoscopy Lateral cervical soft tissue radiograph

Correct Answer ( C ) Explanation: Direct and indirect laryngoscopy both enable visualization of the airway structures, including the epiglottitis. Direct laryngoscopy refers to the use of a laryngoscope to elevate the tongue and supraglottic structures with the patient in a supine position. Indirect laryngoscopy refers to the visualization of the vocal cords without a direct line of sight, using instruments such as a nasopharyngeal laryngoscope. With both methods, however, care should be taken to ensure that manipulation does not lead to laryngospasm and airway obstruction. Prior to any attempts at airway visualization, humidified oxygen should be administered (the data do not support administration of steroids or racemic epi), and emergent intubation along with surgical airway equipment should be gathered. A CT scan of the neck (A) is a very sensitive test; however, it does not have a role in the emergent diagnosis of epiglottitis due to the need for recumbent positioning, travel from the ED, and concern for airway compromise. Lateral cervical soft tissue radiograph (D) has a sensitivity of up to 90% for the diagnosis of epiglottitis. However, a normal-appearing radiograph does not exclude the diagnosis of epiglottitis. Patients with suspected epiglottitis and a normal radiograph should undergo laryngoscopy. A frontal cervical soft tissue radiograph (B) is used to detect subglottic tracheal narrowing in the diagnosis of croup and is of limited utility in the diagnosis of epiglottitis.

Which of the following is the most common location for nosebleeds? Auerbach's plexus Internal carotid artery Kiesselbach's plexus Posterolateral branches of the sphenopalatine artery

Correct Answer ( C ) Explanation: Epistaxis is classified as either anterior or posterior, depending on the location of the bleeding. Anterior nosebleeds are much more common than posterior nosebleeds with up to 90% occurring in Kiesselbach's plexus. Three primary vessels have branches with anastomoses that make up Kiesselbach's plexus: the septal branch of the superior labial branch of the facial artery, the lateral nasal branch of the sphenopalatine artery and the septal branch of the anterior ethmoidal artery. Trauma or irritation of the nasal mucosa are common causes of epistaxis. Auerbach's plexus (A), also known as the myenteric plexus, is located within the muscularis externa. This plexus is made up of nerve cells and is part of the enteric nervous system that regulates peristalsis. Both the internal carotid artery (B) and the posterolateral branches of the sphenopalatine artery (D) are common sources of posterior nosebleeds.

What key clinical finding differentiates erysipelas from cellulitis? Induration Peau d'orange appearance Sharp demarcation from uninvolved skin Systemic symptoms

Correct Answer ( C ) Explanation: Erysipelas is a superficial skin infection involving the upper dermis with prominent lymphatic involvement. Beta-hemolytic streptococci usually cause it. Erysipelas is characterized by an erythematous area of skin that becomes indurated with raised borders distinctly demarcated from the surrounding normal skin. The skin may also exhibit a "peau d'orange" appearance. A classical manifestation is malar involvement with a "butterfly" pattern over the face. Because erysipelas involves the upper dermis and superficial lymphatics, whereas cellulitis involves the deeper dermis and subcutaneous fat, erysipelas can spread to pinna of the ear, whereas cellulitis cannot. Both erysipelas and cellulitis can exhibit induration (A), a "peau d'orange" appearance (B), and systemic symptoms (D).

A 3-year-old boy presents with atraumatic eye pain. Examination reveals a bulging, erythematous, edematous left eye with decreased extraocular movement. Which of the following is the most appropriate initial management step? Consult infectious disease service Consult ophthalmology Intravenous antibiotics Oral antibiotics

Correct Answer ( C ) Explanation: Inflammation behind the orbital septum, the fibrous portions of the eyelids which form the anterior boundary of the orbit, is called orbital cellulitis. Common causes are sinusitis, upper respiratory infections, eye trauma and sepsis. Symptoms include eye pain and limitation with orbital movement, erythema and edema, bulging of the eye and potentially a loss of vision (therefore, it is an ophthalmologic emergency). If untreated, up to 20% of patients will become blind. Inpatient treatment is recommended, and includes hospital admission, labs, cultures, intravenous antibiotics and frequent monitoring with neurologic and vision checks. If improvement is seen in 48 hours, discharge home on oral antibiotics. If no improvement in 48 hours, consider the presence of an abscess and consult surgery and an infectious disease specialist. Ophthalmology (B) and infectious disease (A) consultation should be obtained after initiation of parenteral antibiotics. Outpatient treatment with oral antibiotics (D) is not recommended. Frequent monitoring is necessary as the condition can lead to blindness.

Which of the following is characteristic of myasthenia gravis? Descending paralysis Improvement of muscle strength with repeated stimulation Positive ice bag test Ptosis worse in the morning

Correct Answer ( C ) Explanation: Myasthenia gravis is caused by the autoimmune destruction of the nicotinic acetylcholine receptors on the postsynaptic membrane. As a result of this autoantibody destruction, over time less total receptors are available. At the remaining receptors, the autoantibodies are competing with acetylcholine for the binding sites. With repeated stimulation, fewer receptor sites are both physically available because of destruction and competition, but also because many remain in the refractory period. As a result, muscles weaken and fatigue, which is the hallmark of the disease. Most commonly the first manifestation of the disease is ocular symptoms including ptosis, diplopia and blurred vision. Over the course of the disease, involvement of the respiratory and bulbar muscles may develop. In order to make the diagnosis, 2 options exist. The first is a bedside edrophonium test although the production of this compound is discontinued. Edrophonium is an acetylcholinesterase blocker and therefore causes an accumulation of acetylcholine in the synapse improving symptoms. Symptoms of myasthenia gravis improve with cooling and so the ice bag test is another alternative diagnostic bed-side maneuver. In both scenarios, the distance between the upper and lower eyelids with ptosis is measured. The ice bag test is based on the physiologic principle of improving neuromuscular transmission at lower muscle temperatures, the eyelid muscles are the most easily cooled by the application of ice. After the application of either the ice bag or administration of edrophonium, the distance between the lids is re-measured to assess for a difference. Descending paralysis (A) is classic for botulism in which the botulinum toxin irreversibly binds to presynaptic membranes preventing the release of acetylcholine into the synapse. Botulism occurs in three forms to cause disease in humans: food-borne, wound botulism and infant botulism. Improvement of muscle strength with repeated stimulation (B) is characteristic of Lambert-Eaton syndrome (LES). In LES, antibodies target the calcium channels which results in a reduction of acetylcholine release. With repeated stimulation, more acetylcholine is released from the intact portions of the synapse causing improvement of muscle strength. In MG, ptosis worsens over the course of the day, not in the morning (D).

A 60-year-old man with a history of diabetes presents to your office with a complaint of thickened and discolored toenails. He tells you that his toenails have had this appearance for over a year, and now he is experiencing discomfort when wearing tight-fitting shoes. Physical exam reveals hyperkeratosis and onycholysis of bilateral great and second toes. Which of the following is the most appropriate next step in management? Begin treatment with oral terbinafine Check serum aminotransferases Potassium hydroxide examination of toenail scrapings Watchful waiting

Correct Answer ( C ) Explanation: Onychomycosis is a fungal infection of the toenails or fingernails that can involve any part of the nail including the plate, bed or matrix. There are several subtypes of onychomycosis, with the most common being distal subungual onychomycosis. This type presents with the great toe being the first affected. A white, yellow or brown discoloration can be seen that eventually spreads to the entire nail. Onycholysis, the separation of the nail from the plate, may also be seen. Onychomycosis is initially a cosmetic concern, however with time it can cause pain, disfigurement, and decreased quality of life. Other nail dystrophies can present similarly to onychomycosis, therefore establishing the presence of a fungal etiology is recommended prior to initiation of treatment. Diagnosis is with potassium hydroxide (KOH) examination of nail scrapings. Patients who are immunocompromised or who have diabetes mellitus are at an increased risk of bacterial infections due to onychomycosis. Treatment should be considered in these patients to avoid sequelae. Once a fungal etiology has been determined, first-line treatment is with oral antimycotic agents such as terbinafine (A). Terbinafine can cause hepatotoxicity, so pretreatment serum aminotransferases (B) should be measured prior to initiating therapy and then monitored during the course of treatment. Watchful waiting (D) may be appropriate in patients without risk factors. Patients with diabetes should be treated to avoid complications.

Which of the following conditions is strongly associated with optic neuritis? Herpes zoster Hyperthyroidism Multiple sclerosis Myasthenia gravis

Correct Answer ( C ) Explanation: Optic neuritis is a common presenting symptom of multiple sclerosis (MS). Approximately 75% of women and 34% of men with optic neuritis will develop MS within 15 years of the onset of optic neuritis. Optic neuritis is localized inflammation of the optic nerve sheath resulting in reduced neuronal transmission and decreased visual acuity. Generally, there is a loss of color vision and red desaturation noticed by the patients. On exam, there is an associated afferent pupillary defect and funduscopic exam reveals papillitis and swelling or edema of the optic nerve head (but may be normal in retrobulbar optic neuritis). The symptoms generally worsen during the first few days and progressively improve over several weeks. In children, optic neuritis has various causes, but is typically associated with viral infections. The workup include MR neuroimaging to investigate for plaque formations secondary to multiple sclerosis. Intravenous methylprednisolone (1 g/day for 3 days) followed by oral prednisone (1 mg/kg/day for 11 days) reduced the rate of MS development for 2 years. Corticosteroids hasten visual recovery; administration of steroids does not affect the overall visual acuity. Herpes zoster (A) eye findings include corneal dendrites and ulcers and can occur with herpes simplex virus. Hyperthyroidism (B) may induce an orbitopathy in some patients. In such cases, there is diffuse hyperplasia of the thyroid and infiltrative ophthalmopathy. Myasthenia gravis (D) eye findings include unilateral or bilateral ptosis, which is usually asymmetric, along with lid fasciculations, and lid retraction.

An uncircumcised 2-year-old boy presents with penis pain. Examination reveals a swollen, nonreducible foreskin which compresses the penis, resulting in glans erythema and edema. Based on these physical examination findings, which of the following is most likely diagnosis? Balanitis Hypospadia Paraphimosis Posthitis

Correct Answer ( C ) Explanation: Paraphimosis is foreskin that, once retracted, cannot be brought to usual position, resulting in constriction. If this persists for several hours, the continued retraction can cause a strangulation effect on the glans arterial blood supply, leading to ischemia and possible gangrene. As such, this is considered a urologic emergency. Causes include improper handling of the foreskin, as in repeated or prolonged retraction during cleaning, examination, catheterization or cystoscopy, infection and pathologic phimosis. Symptoms include inability to reduce the foreskin, foreskin erythema and edema, glans erythema and edema and pain. Paraphimosis can be prevented by returning the foreskin back to its flaccid position after retraction is no longer necessary. Treatment includes manual manipulation of the swollen foreskin back into its flaccid position, which may require the use of lubricants, cold compression or anesthetics. If unsuccessful, an emergent dorsal slit procedure or Dundee procedure may be tried. Balanitis (A) is inflammation of the glans penis typically not associated with foreskin entrapment and strangulation. Causes include infection (STI), environmental sensitivities and physical trauma. When the glans penis is erythematous and edematous due to a nonretractable foreskin, the condition is more properly called paraphimosis and not balanitis. Hypospadia (B) occurs when the fusion of the urethral folds stops proximal to the distal end of the glans penis, resulting in a ventral urinary meatus. It is usually asymptomatic and not associated with erythema or edema. Posthitis (D) is inflammation of the foreskin only, commonly associated with diabetes mellitus, STIs, contact dermatitis and psoriasis. There is no associated strangulation of the glans.

An 11-year-old boy presents due to concern for noisy breathing. Over the past three days, he complained of a sore throat and a fever. On examination, he is in mild distress and has prominent halitosis. His voice is muffled. Oropharyngeal examination is limited by trismus, but the visualized portion of the pharynx reveals exudate over both tonsils. The left tonsil is 1+, and the right is 4+. The uvula is deviated to the left. He has mild subcostal retractions and soft inspiratory stridor, but the remainder of his respiratory examination is normal. What is the most likely cause of his stridor? Epiglottitis Laryngotracheitis Peritonsillar abscess Retropharyngeal abscess

Correct Answer ( C ) Explanation: Peritonsillar abscesses are most common among children older than 10 years of age, when Streptococcal pharyngitis is more common. Peritonsillar abscesses are typically preceded by pharyngitis and tonsillitis prior to abscess formation. Abscess formation is characterized by severe sore throat, which is classically asymmetric, as well as a muffled "hot potato" voice. Stridor, drooling, trismus, and neck swelling may also be present. On examination, the abscess may bulge anteriorly over the soft palate and be overtly fluctuant. Alternatively, the patient may simply have asymmetric tonsillar swelling with deviation of the uvula toward the unaffected side. Epiglottitis (A) is swelling of the epiglottitis that is classically caused by Haemophilus influenzae type B (HIB). It classically presents with rapid progression of respiratory distress, drooling, and dysphagia. The above patient's subacute progression of symptoms is less severe than that of epiglottitis. Additionally, epiglottitis would not cause exudative tonsillitis or asymmetric tonsillitis. Laryngotracheitis (B), also known as croup, is the most common cause of stridor in children. Croup is most common amongst children 6 to 36 months of age. The onset of stridor is preceded by several days of upper respiratory symptoms, such as nasal congestion and low grade fevers. The above child has neither the classic age or symptom onset of croup. Additionally, viral croup does not cause an exudative pharyngitis or asymmetric tonsillar enlargement. Retropharyngeal abscesses (D) are most common amongst children aged two to four years. Initially the child will appear to have pharyngitis prior to symptom progression to fever, drooling, stiff neck, a muffled voice, and inspiratory stridor. The above patient's age and asymmetric tonsils make a peritonsillar abscess more likely than a retropharyngeal abscess.

You are caring for a 60-kg patient who has been seizing for 30 minutes. You have already administered 4 mg of IV lorazepam and 1200 mg of phenytoin without termination of seizure activity. Which of the following should most likely be your next step? Administer another bolus of IV phenytoin Administer IV fosphenytoin Administer IV pentobarbital Administer IV sodium bicarbonate

Correct Answer ( C ) Explanation: Seizing patients become apneic and may vomit, compromising their airway and putting them at significant risk for hypoxia and aspiration. Failure to respond to optimal benzodiazepine and phenytoin loading functionally defines refractory status epilepticus. At this point, a third line agent such as pentobarbital (15 mg/kg IV) should be administered. Use caution when adding barbiturates to benzodiazepines because their co-administration may potentiate respiratory depression. Propofol is an alternative to pentobarbital and has the advantages of a short half-life and rapid clearance. These agents are given by IV drip and titrated to a burst-suppression pattern in the EEG tracing. Consider also other causes of benzodiazepine-resistant seizures such as isoniazid toxicity (treated with pyridoxine) and hyponatremia. Phenytoin (A) dosing is 20-30 mg/kg. This patient has already received a 20 mg/kg bolus. Additionally, phenytoin can lead to toxicity from its diluent propylene glycol. It is best to use another agent. Fosphenytoin (B) is a prodrug of phenytoin that can be infused at a faster rate. However, it is prudent not to administer in the setting of full-dose phenytoin due to the risk of toxicity. Although most patients in refractory status epilepticus will have a metabolic acidosis, sodium bicarbonate (D) does not play a role in treating seizures.

A four-year-old girl is seen at the ED because of possible ingestion. She was unattended for several minutes when she went through her grandmother's bag containing bottles of bethanecol, clonidine, diphenhydramine, and fluoxetine. Upon arrival at the ED, her examination reveals temperature of 38°C, heart rate of 120, respiratory rate of 16, blood pressure 85/56, pupils 5 mm equal and reactive and dry skin. Which of the following is the most likely medication that caused the girl's findings? Bethanecol Clonidine Diphenhydramine Fluoxetine

Correct Answer ( C ) Explanation: The girl has clinical findings of anticholinergic poisoning that consists of hyperthermia, tachycardia, mydriasis, and dry skin. Other symptoms include decreased bowel sounds, urinary retention, disorientation, bizarre behavior, paranoia, delirium, visual hallucinations, and, in severe cases, seizures. The classic description of anticholinergic intoxication is known as: "red as a beet," "dry as a bone," "hot as a hare," "blind as a bat," and "mad as a hatter." Diphenhydramine is a medication with anticholinergic effects, and this is the medication the girl in the vignette most likely ingested. Examples of classes of medications with anticholinergic properties include antihistamines, tricyclic antidepressants, sleep aids and cold preparations. Many plants, such as jimson weed (Datura stramonium) and deadly nightshade (Atropa belladonna), may also produce anticholinergic toxicity. Initial management begins with stabilization of the airway, breathing, and circulation. Most patients do well with supportive care alone, but some may benefit from antidotal therapy with physostigmine. Bethanecol (A) has cholinergic properties, and intoxication manifests as diarrhea, urination, miosis, bronchospasm, emesis, lacrimation, salivation, lethargy, wheezing, and bradycardia. Clonidine (B) ingestion would reveal lethargy, and miosis, and bradycardia. In severe cases, there may be hypotension, respiratory depression, and apnea. Fluoxetine (D) is a selective serotonin reuptake inhibitor (SSRI) and manifestations of toxicity include sedation and tachycardia. In significant overdoses, there may be cardiac conduction abnormalities (primarily QTc prolongation) and seizures.

A 17-year-old girl presents with the above rash a week after hiking in the woods. She does not report a tick bite. She otherwise has no symptoms. What management is indicated? Ceftriaxone 2 grams for 14 days Chloramphenicol 1 gram for 21 days Doxycycline 100 mg twice a day for 3 weeks No treatment while awaiting diagnostic testing

Correct Answer ( C ) Explanation: This patient presents with erythema migrans, the typical rash seen in early Lyme disease and requires 3 weeks of treatment with oral doxycycline. Lyme disease is an illness caused by transmission of Borrelia burgdorferi bacteria from a tick. The common vector is Ixodes scapularis. Transmission from tick to person requires attachment and feeding for more than 48 hours. Thus, early removal of ticks can prevent transmission. Typically, erythema migrans presents 7 to 10 days after infection and is proceeded by a non-specific constitutional symptoms (fever, malaise, fatigue etc.). Approximately 90% of patients report the presence of the rash. The rash begins as a small papule at the site of infection and gradually expands (1-2 cm/day). Typically, the rash will have central clearing but this is not universal. Further hematogenous spread of B. burgdorferi can cause numerous symptoms including arthralgias, neurologic manifestations, heart block etc. The diagnosis of Lyme disease is based primarily on clinical features but serologic or ELISA testing can be used to confirm the diagnosis. Prompt treatment of early manifestations can both shorten symptom duration and prevent progression to later disease stages. Early Lyme disease should be treated with oral doxycycline 100 mg twice a day for 21 days. Amoxicillin and cefuroxime are alternatives to doxycycline. You should also make sure that you obtain a negative pregnancy test in all females of child bearing age before starting on doxycycline. Ceftriaxone (A) is a third generation cephalosporin that should be used in patients with Lyme meningitis, moderate to severe cardiac manifestations or arthritis. Chloramphenicol (B) is an alternative to ceftriaxone in the treatment of Lyme meningitis. Withholding treatment while waiting for diagnostic testing (D) is not recommended because the diagnosis should be made clinically and both serologic and ELISA tests suffer from less than perfect sensitivity and specificity.

A 15-year-old boy presents with decreased hearing and otorrhea. On examination of the ear, you see the image above. Which of the following is safe? Antipyrine-benzocaine drops Gentamicin drops Ofloxacin otic suspension Swimming

Correct Answer ( C ) Explanation: Tympanic membrane perforations occur most commonly from infections as pressure builds up behind the membrane causing it to rupture. Perforation may also result from trauma including direct ear trauma, explosions, severe pressure (diving) and direct injury from ear cleaning or other instrumentation. In the setting of infection, copious purulent drainage in the external canal is often present. Most perforated membranes heal spontaneously without intervention. Antibiotics may be considered in the presence of an active infection. Typically the suspension form is used rather than solution due to increased viscosity and less likelihood of entering the inner ear. This may be given in combination with antibiotic drops although prophylactic antibiotics for an isolated perforation are not required. Rarely, ENT physicians may place a patch over the eardrum if healing is prolonged or ultimately repair the perforation surgically. In children with a perforated tympanic membrane after otitis media, systemic oral therapy is preferred over topical agents. The quinolones (e.g. ciprofloxacin or ofloxacin) are not toxic and can be administered to patients with perforated tympanic membranes. Antipyrine-benzocaine drops (A) are used for pain control in patients with otitis media. However, when the tympanic membrane is perforated the benzocaine portion is ototoxic and should be avoided. Gentamicin drops (B) are a member of the aminoglycoside family of antibiotics and similarly have ototoxic effects and are contraindicated. One of the most important components of treatment of a perforation is keeping the ear canal dry. Therefore, swimming (D) should be avoided. Additionally, patients should be advised to wear a shower cap when showering to prevent water from entering the canal.

A 65-year-old man complains of sudden pain and vision loss in his left eye about 20 minutes after entering a movie theater. Which of the following is the most appropriate treatment for his condition? Cyclopentolate Digital massage of the globe High-dose steroid Topical beta-blocker

Correct Answer ( D ) Explanation: Acute angle glaucoma can be triggered by dim light (such as going from bright light outside to a darkened theater), which dilates the pupil leading to occlusion of the trabecular meshwork and obstruction to aqueous humor outflow resulting in increased intraocular pressure. Treatment is aimed at rapid reduction of intraocular pressure by inhibiting production of aqueous humor and improving outflow. This can be achieved with a topical beta-blocking agent such as timolol. Additional medications that may be used include alpha-agonists (brimonidine), prostaglandin analogues (latanoprost), and pilocarpine (topical miotic). Adjunctive therapy includes carbonic anhydrase inhibitors (acetazolamide) and osmotic diuretics (mannitol). Corneal indentation is reserved for intraocular pressures >50 mm Hg or when other agents are ineffective. Cyclopentolate (A) is a mydriatic agent that can induce and worsen acute angle glaucoma. As such, it should be avoided in all patients at risk for this condition. Digital massage of the globe (B) is a technique used in patients with central retinal artery occlusion to help dislodge the embolus. High-dose steroids (C) are used in patients with vision loss due to temporal arteritis.

A 16-year-old sexually active girl presents to your clinic with foul-smelling vaginal discharge. Examination of the vaginal fluid reveals clue cells. The vaginal pH is 5. Which of the following is the best course of treatment for this condition? Azithromycin Ceftriaxone Doxycline Metronidazole

Correct Answer ( D ) Explanation: Bacterial vaginosis (BV) presents with a thin, white-to-gray vaginal discharge that has a "fishy" odor when a drop of 10% KOH is added to fluid. Symptoms of inflammation are usually absent. Dysuria and dyspareunia are rarely present. The diagnosis is generally made with the presence of foul-smelling discharge, vaginal pH greater than 4.5 and greater than 20 percent clue cells present on saline wet mount. Gardnerella vaginalis is associated with BV, but other organisms are also involved, including Bacteroides species and Mobiluncus species. Gardnerella vaginalis can colonize the vagina and therefore, a bacterial culture would not be useful in the diagnosis. Treatment in non-pregnant women is metronidazole 500 mg twice a day for seven days. An intravaginal metronidazole gel can also be used for five days. Treatment results can vary as cure rates are not 100 percent; however, a test of cure is not necessary. Azithromycin (A) is macrolide that is not used in the treatment of bacterial vaginosis. Ceftriaxone (B), a third-generation cephalosporin, is administered parenterally and not appropriate for outpatient therapy. Doxycyline (C) is used in combination with ceftriaxone to treat gonorrhea and chlamydia but has no role in the treatment for BV.

Which of the following is the most common type of cervical cancer? Adenocarcinoma Cervical sarcoma Small cell carcinoma Squamous cell carcinoma

Correct Answer ( D ) Explanation: Cervical cancer is the third most common gynecologic cancer in the United States with a lower rate of incidence and mortality than ovarian or uterine cancer. Rates are lower because of the use of Papanicoloau (Pap) screening which allows for detection of pre-cancerous and early-stage disease. Countries worldwide without access to cervical cancer screening have rates of cervical cancer that are much higher than the United States, with cervical cancer remaining the second most common type of cancer in women worldwide. Human papillomavirus is found in more than 99% of all cervical cancers and causes the development of cervical neoplasia. Risk factors for cervical cancer include activities that lead to infection with the human papillomavirus (HPV), oral contraceptive use and cigarette smoking. The most common type of cervical cancer is squamous cell carcinoma, which is found in 69% of cases of cervical cancer in the United States. Adenocarcinoma (A) is the second most common type of cervical cancer. It is found in 25% of cases of cervical cancer in the United States. Other histologies make up 6% of cases of cervical cancer including cervical sarcoma (B) and small cell carcinoma (C).

A mom brings in her 16-year-old daughter, with concerns of increasing weight loss. Over the past 3 months, her weight went from 60 kg to 55 kg, placing her at the 10th percentile for weight and BMI. She states she is just very conscience of her food choices and wants a regimented exercise routine, as she is training for High School track. She does admit to irregular scant menstrual periods. What constellation of signs and symptoms should you be concerned about in this patient? Anemia, bradycardia, osteopenia Anemia, easy bruising, multiple fractures Disordered eating, amenorrhea, abnormal body image Disordered eating, amenorrhea, osteopenia

Correct Answer ( D ) Explanation: Disordered eating, amenorrhea, and osteopenia is known as the female athlete triad. This is the most likely concern in this patient who presents with a history of weight loss, irregular periods, intense athletic training, and a restricted regimen of food intake and exercise. Disordered eating is related to the need to maintain the lean body mass desired in high endurance sports and, depending on how severe the eating disorder is, may be seen in more than half of female athletes. Amenorrhea can range from oligomenorrhea to primary or secondary amenorrhea, and is thought to result from suppression of gonadotropin releasing hormone, as a result of imbalanced energy expenditure, and intake. Osteopenia results from a disruption in bone resorption, bone formation, or a combination of both. Anemia (A) can be a considerable component in individuals with unhealthy dietary restrictions. Individuals with low body mass such as anorexic individuals may also present with bradycardia. However, these are not a part of the female athlete triad. Easy bruising (B) is related to a platelet disorder or coagulopathy and is not directly related to the triad. There is an increased risk for bone fractures due to osteopenia; however multiple fractures are not commonly seen. Abnormal body image (C) can definitely become a part of the abnormal behavior found in some female athletes however, this is also not a part of the triad.

A 45-year-old woman presents to your office with a complaint of pain "all over." She tells you that over the past few years her pain has worsened and she also experiences daily fatigue, difficulty concentrating, and headaches. Physical exam reveals multiple points of tenderness to palpation. Which of the following lab results would be expected? Decreased vitamin B12 Elevated erythrocyte sedimentation rate Elevated thyroid stimulating hormone Normal complete blood count

Correct Answer ( D ) Explanation: Fibromyalgia is a syndrome that causes chronic, widespread pain and tenderness. The etiology of fibromyalgia is unknown and the pathogenesis has not been proven, however it is believed to be related to a dysfunction in central pain processing. In addition to complaints of pain, patients often present with fatigue, cognitive difficulties, and multiple somatic and psychiatric symptoms. Fibromyalgia is a diagnosis of exclusion and patients should be evaluated for other disorders before making the diagnosis. Physical exam findings are generally within normal limits other than tenderness to palpation of multiple soft tissue points. Laboratory test results, including complete blood count, are within normal limits. There is no cure for fibromyalgia and management involves a multi-disciplinary approach including education, lifestyle modifications and medication. Patients should be counseled regarding proper diet, physical exercise, stress reduction, and healthy sleep habits. An initial pharmacologic approach with monotherapy using amitriptyline, duloxetine, pregabalin or milnacipran is recommended. Prognosis for patients with fibromyalgia varies, with better outcomes seen when patients seek help, engage in more physical exercise, pace their activities and in those that have less guarding during physical exam. Laboratory testing in patients with fibromyalgia is generally normal and is used to rule out disorders with symptoms that mimic fibromyalgia. Decreased vitamin B12 levels (A) can cause fatigue and pain. Elevated erythrocyte sedimentation rate (B) indicates the presence of an inflammatory process. Elevated thyroid stimulating hormone (C) is seen in patients with hypothyroidism which can present with symptoms of fatigue and diffuse muscle pain.

A 45-year-old woman with a history of Grave's disease presents to the ED with a 1-day history of dyspnea, fever, and agitation. Her husband believes that she stopped taking her thyroid medication 1 week ago. Her vital signs are BP 185/75, HR 141, R 24, and T 38.9°C. On exam, you note tremulous hands and bounding peripheral pulses. Her ECG shows a sinus tachycardia; chest radiograph shows increased interstitial markings. You administer 2 L of normal saline, but her vital signs are unchanged. Which of the following is the most appropriate therapy to administer next? Cefepime Dantrolene Potassium iodide Propranolol

Correct Answer ( D ) Explanation: Hyperthyroidism refers to disorders that result from overproduction of hormone from the thyroid gland. Thyrotoxicosis refers to any cause of excessive thyroid hormone concentration. Thyroid storm represents the extreme manifestation of thyrotoxicosis. The point at which thyrotoxicosis transforms to thyroid storm is controversial, but attempts have been made by Burch and Wartofsky to standardize the diagnosis of thyroid storm. This patient has thyroid storm, a condition predominantly seen in patients with Grave's disease. There are many precipitants of thyroid storm, but infection and sepsis are the most common. Clinically, patients often present with fever, diaphoresis, tachycardia, altered mental status, restlessness, agitation, abdominal pain, and vomiting. The presentation mimics many other hyperadrenergic states such as cocaine intoxication or ethanol withdrawal. It can also be confused with heatstroke and neuroleptic malignant syndrome. The prompt recognition and treatment of thyroid storm is crucial for patient survival. If untreated, the condition is uniformly fatal. Propranolol is the first-line treatment for patients with symptomatic thyrotoxicosis or thyroid storm because it blocks peripheral hyperadrenergic activity in addition to the conversion of T4 to T3. This should be immediately followed by administraiton of propylthiouracil to further block the conversion of T4 to T3. Iodine (after PTU administration), corticosteroids, fluid resuscitation, rapid cooling, and treatment of precipitating illness follows. After administration of propranolol, management of thyroid storm should proceed with the initiation of propylthiouracil to inhibit the production of thyroid hormone. Once the primary measures are taken, and there is concern that the inciting event may be due to sepsis, then cefepime (A) should be administered. Inorganic iodine (potassium iodide) (C) blocks the release of thyroid hormone that is still stored in the thyroid gland, but its administration should be delayed at least 60 min after propylthiouracil is administered. It is important to first inhibit the synthesis of thyroid hormone; otherwise, the administered iodine will be incorporated into new hormone. Dantrolene (B) is a muscle relaxant that acts by abolishing excitation-contraction coupling in muscle cells, probably by action on the ryanodine receptor. It is the only specific and effective treatment for malignant hyperthermia, a rare, life-threatening disorder triggered by general anesthesia.

A 45-year-old Caucasian man presents to your office with questions about prostate cancer screening. He does not have a family history of prostate cancer and wants to know at what age he should begin screening. Which of the following is the most appropriate next step in management? Begin screening at age 50 Order serum prostate specific antigen now, then begin screening at age 50 Perform an initial digital rectal exam now, then begin screening at age 50 Screening is not recommended

Correct Answer ( D ) Explanation: In the United States, prostate cancer is the most frequently diagnosed type of cancer in men after skin cancer. Prostate cancer is seen more commonly in African-American men and the likelihood of developing this type of cancer increases with age. Risk factors include a family history of prostate cancer, cigarette smoking and a diet high in animal fat. Routine screening for prostate cancer has been a controversial subject. Current United States Preventive Services Task Force guidelines recommend against prostate specific antigen (PSA) screening for prostate cancer. Some men will request screening and a discussion about the risks and benefits of screening should occur prior to initiating PSA testing or digital rectal exam (DRE). Most patients diagnosed with prostate cancer are asymptomatic and the cancer is found on DRE or because of an elevated serum PSA. Diagnosis is made with biopsy. Treatment decisions are determined after discussion with the patient about the severity of disease and quality of life related to treatment side effects. Options include active surveillance, prostatectomy, radiation therapy and hormonal therapy. Discussion regarding screening for men at average risk should begin at age 50 years (A), unless their life expectancy is less than 10 years. Concerns exist about the benefits of screening versus the potential harm to quality of life due to overdiagnosis and treatment complications, which is why universal screening is not recommended. Prostate cancer screening is done using serum PSA (B) and DRE (C). Discussion must occur first and once the decision has been made to screen, it is with both techniques.

On fundoscopy you identify a bulging, erythematous optic disk. Visual field, acuity and pupillary light reflex testing is normal. Which of the following is the most likely diagnosis? Anterior uveitis Blepharitis Optic neuritis Papilledema

Correct Answer ( D ) Explanation: Papilledema, optic disk edema, is due to increased intracranial pressure, which can be due to intracranial hypertension, tumors or hemorrhage. In contrast to other causes of optic disc swelling, vision usually is well preserved with acute papilledema. Papilledema almost always presents as a bilateral phenomenon and may develop over hours to weeks. Anterior uveitis (A), also known as iritis, is inflammation of the choroid and iris that causes deep pain, blurred vision and erythema. The optic disk, a posterior structure, is usually normal appearing. Blepharitis (B) is eyelid inflammation, not optic disk edema and erythema. Optic neuritis (C) usually presents with diminished acuity and an abnormal pupillary light reflex.

A 42-year-old woman presents to your office with complaints of fatigue, pruritus and abdominal discomfort. Laboratory testing reveals elevated serum alkaline phosphatase and hyperlipidemia. Physical exam findings include jaundice, skin excoriations and hepatomegaly. Which of the following is the most likely diagnosis? Acute pancreatitis Pancreatic cancer Polycythemia vera Primary biliary cirrhosis

Correct Answer ( D ) Explanation: Primary biliary cirrhosis is a rare disease of the liver that causes progressive cholestasis and can result in end-stage liver disease. It is believed to be autoimmune in nature, with T-lymphocytes attacking the bile duct epithelial cells, leading to their destruction. The loss of these bile ducts causes the signs and symptoms of cholestasis. Patients may be asymptomatic or present with fatigue, pruritus and right upper quadrant discomfort. Physical exam findings include skin hyperpigmentation, excoriations, jaundice, and hepatosplenomegaly. Physical manifestations of liver cirrhosis such as spider nevi, ascites and muscle wasting may be seen in later stages of the disease. Laboratory abnormalities include elevated alkaline phosphatase, antimitochondrial antibodies, antinuclear antibodies and hyperlipidemia. Diagnosis is determined when the patient has no comorbidities affecting the liver, no extrahepatic biliary obstruction and has at least two of the following: evidence on histology, the presence of antimitochondiral antibodies, and elevated alkaline phosphatase at least 1.5 times the normal limit. Treatment goals are to alleviate symptoms and slow progression of the disease. The only life-saving procedure available is liver transplantation. Patients with acute pancreatitis (A) experience severe abdominal pain, nausea and vomiting. Laboratory findings include elevated serum amylase and lipase. Patients with pancreatic cancer (B) may present with fatigue, weight loss, abdominal pain, jaundice and hepatomegaly. Pruritus is not commonly seen in pancreatic cancer. Patients with symptoms suspicious for pancreatic cancer should have initial diagnostic imaging done with transabdominal ultrasound. Polycythemia vera (C) causes aquagenic pruritus, burning pain in hands and feet, venous or arterial thromboses, transient visual disturbances and gastrointestinal complaints. Laboratory testing results include elevated hemoglobin, hematocrit, red blood cell mass and vitamin B12.

Which of the following is the most common sign seen in patients presenting with a pulmonary embolism? Hemoptysis Hypoxia Swollen calf Tachypnea

Correct Answer ( D ) Explanation: Tachypnea is the most common vital sign abnormality seen in acute pulmonary embolism (PE). PE is a common, life-threatening disease caused by the occlusion of pulmonary arteries by blood clots. The majority of these clots originate in the deep venous system (more commonly in the legs than in the upper body). Patients can present with various symptoms including weakness, shortness of breath, malaise, syncope, dizziness or chest pain. In addition, PE can cause changes to all of the major vital signs. Sinus tachycardia is the most common abnormality found on ECG. Hypoxia (B) and hemoptysis (A) are common in larger PE but are frequently not seen in smaller ones. While a swollen calf (C) may be indicative of a possible PE, it is not the most common finding.

Which of the following is the most common etiology of external otitis? Aspergillus niger Candida Moraxella catarrhalis Pseudomonas aeruginosa

Correct Answer ( D ) Explanation: The most common cause of otitis externa is infection due to Pseudomonas aeruginosa and Staphylococcus aureus. The pain from otitis externa is caused by inflammation and edema of the ear canal skin, which is normally adherent to the bone and cartilage of the auditory canal. The inflammatory reaction can be caused by bacteria, fungi, or contact dermatitis. Cerumen protects the canal by forming an acidic coat that helps prevent infection. Factors that predispose to otitis externa include absence of cerumen, often from excessive cleaning by the patient, water that macerates the skin of the auditory canal and raises the pH, and trauma to the skin of the auditory canal from foreign bodies or use of cotton swabs. Treatment includes suction and gentle warm irrigation of the canal. 2% acetic acid solution or an alternative drying medication can be administered. A topical antibiotic drop with steroid is first-line therapy (neomycin/polymyxin/hydrocortisone). Use the suspension rather than the solution if the tympanic membrane is ruptured. Fungal infections compose less than 10% of external otitis cases. The most common fungi are Aspergillus niger (A) and Candida (B) species and they are more prevalent in tropical climates. Moraxella catarrhalis (C) is a common contributor to otitis media but is not often implicated in otitis externa.

An 18-year-old woman presents with dysuria, hematuria, fever and flank pain. Physical examination reveals a well-appearing woman with moderate costovertebral angle tenderness. Urinalysis is positive for leukocyte esterase, nitrites and 25-50 WBCs. What treatment is indicated? Intravenous ceftriaxone Intravenous ciprofloxacin Nitrofurantoin Oral trimethoprim-sulfamethoxazole (TMP-SMX)

Correct Answer ( D ) Explanation: The patient presents with uncomplicated pyelonephritis, which can be treated as an outpatient with trimethoprim-sulfamethoxazole (TMP-SMX). Urinary tract infection (UTI) is a common problem affecting all age groups and is more common in women than in men. Patients with upper UTI or pyelonephritis often present with fever, malaise, nausea, vomiting, back and flank pain in addition to the dysuria, frequency and hematuria seen in lower UTI. Urinalysis in UTI typically shows leukocyte esterase and nitrites. Microscopy will show at least 10 WBCs. Escherichia coli is the most common bacteria causing UTI, particularly in outpatients. Institutionalized or hospitalized patients may have involvement with atypical bacteria like Proteus, Klebsiella and Enterobacter. Treatment must be based on whether the patient has a simple or complicated infection. Complicated UTI involves comorbidities, pregnancy, toxic appearance, immunocompromise, vital sign abnormalities and inability to tolerate oral intake. Those with simple pyelonephritis may be managed as outpatients with appropriate antibiotics including TMP-SMX (when resistance is <10%), fluoroquinolones or late-generation cephalosporins are alternate options. Intravenous ceftriaxone (A) and ciprofloxacin (B) are appropriate inpatient treatment for pyelonephritis or complicated UTI. Nitrofurantoin (C) is appropriate for the treatment of cystitis but is not active in upper UTI.


Kaugnay na mga set ng pag-aaral

Medical Terminology: Ch. 3/Abdominopelvic Regions and Quadrants

View Set

MANAGEMENT OF THE PATIENT WHO HAS CANCER QUESTIONS

View Set

LIFE ONLY_Chapter 7-Qualified Plans

View Set

CS fundamentals networking questions

View Set

Fundamentals of Computer Science Final Exam Review

View Set